Cannon Projectile Motion Problem

In summary, the conversation discusses a problem involving projectile motion, specifically solving for the initial velocity needed to hit a target given the angle of projection and other known variables. The conversation includes attempts at solving the problem using various equations and formulas, as well as possible mistakes made in the process. Ultimately, the final equation is found to be incorrect due to a missing negative sign in one of the variables.
  • #1
vanquish
35
0
[SOLVED] Projectile Motion

Homework Statement


You are shooting a cannon from a height of 11.23m off the ground. The target is 20.36m away. The angle of projection is 30 degrees. What does the initial speed need to be in order to hit the target?


Homework Equations


When I look at this problem I just cannot solve it. I personally don't think I have enough information but it has really been bugging me.


The Attempt at a Solution


I know I need to figure out one of the velocities, either for the x or the y axes in order to solve for the initial velocity. The only information I have is:
[tex]\Delta[/tex]y=11.23
a=-10

[tex]\Delta[/tex]x=20.36
a=0

I tried using the range formula before remembering that it can only be used when the initial and final velocities are the same.
 
Physics news on Phys.org
  • #2
You have everything you need.

[tex]x=(v_0\cos\theta)t[/tex]
[tex]y=(v_0\sin\theta)t-\frac 1 2 gt^2[/tex]

Solve for t in the x and plug it into y, then solve for v initial.
 
  • #3
Thanks very much, that makes perfect sense. I'll keep a lookout for problems like these in the future
 
  • #4
vanquish said:
Thanks very much, that makes perfect sense. I'll keep a lookout for problems like these in the future
Welcome. You may also want to write those formulas down in your book for future reference so you don't have to go through the annoying algebra everytime.
 
  • #5
rocophysics said:
You have everything you need.

[tex]x=(v_0\cos\theta)t[/tex]
[tex]y=(v_0\sin\theta)t-\frac 1 2 gt^2[/tex]

Solve for t in the x and plug it into y, then solve for v initial.

I just tried it and the answers does not make sense.
 
  • #6
Yea said:
I just tried it and the answers does not make sense.
What did you get?
 
  • #7
I got vo= 72
 
  • #8
Yea said:
I got vo= 72
Final equation should be ...

[tex]v_0=\frac{x}{\cos\theta}\sqrt{\frac{g}{2(x\tan\theta-y)}}[/tex]
 
  • #9
I got the same problem... maybe our algebra is the problem
 
  • #10
[tex]y=v_0\sin\theta\left(\frac{x}{v_0\cos\theta}\right)-\frac{1}{2}g\left(\frac{x}{v_0\cos\theta}\right)^2[/tex]

[tex]y=v_0\tan\theta-\frac{gx^2}{2(v_0\cos\theta)^2}[/tex]
 
Last edited:
  • #11
rocophysics said:
Final equation should be ...

[tex]v_0=\frac{20.36}{\cos\(30)}\sqrt{\frac{-10}{2(20.36\tan\(30)-11.23)}}[/tex]

This is the equation with all the numbers plugged in, and its still not turning out right

(should be cos 30 and tan 30, but i couldn't get it to work)
 
  • #12
what is up with this problem, why doesn't the answer make any sense?
 
  • #13
The algebra looks all right, so I don't understand how we could be getting such a wrong answer. I have a feeling it's a simple little mistake
 
  • #14
i have a feeling that this problem cannot be that complicated.
 
  • #15
vanquish said:
The algebra looks all right, so I don't understand how we could be getting such a wrong answer. I have a feeling it's a simple little mistake
What's the actual answer?
 
  • #16
we forgot the negative on the delta y...
 
  • #17
ahhhh that's the key
 
  • #18
vanquish said:
we forgot the negative on the delta y...
ahh that's the key
 
  • #19
vanquish said:
we forgot the negative on the delta y...
You're the man or woman! :-]
 

1. What is the definition of "Cannon Projectile Motion Problem"?

The Cannon Projectile Motion Problem is a physics problem that involves calculating the motion of a projectile fired from a cannon, taking into account factors such as initial velocity, angle of launch, and air resistance.

2. What is the formula for calculating the range of a projectile in the Cannon Projectile Motion Problem?

The formula for calculating the range of a projectile in the Cannon Projectile Motion Problem is: R = (v^2 * sin(2θ))/g, where R is the range, v is the initial velocity, θ is the angle of launch, and g is the acceleration due to gravity.

3. How does air resistance affect the trajectory of a projectile in the Cannon Projectile Motion Problem?

Air resistance, also known as drag, acts in the opposite direction of the projectile's motion and decreases its velocity. This results in a decrease in the range and height of the projectile, and a change in its trajectory.

4. What is the optimal angle of launch for achieving maximum range in the Cannon Projectile Motion Problem?

The optimal angle of launch for achieving maximum range in the Cannon Projectile Motion Problem is 45 degrees. This angle results in the highest range for a given initial velocity, assuming no air resistance.

5. Can the Cannon Projectile Motion Problem be solved without using calculus?

Yes, the Cannon Projectile Motion Problem can be solved without using calculus. The problem can be solved using the basic equations of motion, such as the formula for range and the equations for horizontal and vertical motion. However, using calculus can provide a more precise and comprehensive solution.

Similar threads

  • Introductory Physics Homework Help
Replies
11
Views
762
  • Introductory Physics Homework Help
Replies
15
Views
1K
Replies
2
Views
2K
  • Introductory Physics Homework Help
Replies
4
Views
1K
  • Engineering and Comp Sci Homework Help
Replies
1
Views
250
  • Introductory Physics Homework Help
Replies
19
Views
1K
  • Introductory Physics Homework Help
Replies
6
Views
186
  • Introductory Physics Homework Help
Replies
7
Views
3K
  • Introductory Physics Homework Help
Replies
6
Views
1K
  • Introductory Physics Homework Help
Replies
21
Views
2K
Back
Top